Conjectura relacionada à Lei Kolmogorov 0-1 (para eventos)


8

Seja um espaço de probabilidade. Conjetura:(Ω,F,P)

Suponha que tenhamos os eventos st , ou . Existe uma sequência independente de eventos stA1,A2,... Anσ(An,An+1,...)P(A)=01B1,B2,...

τAn:=nσ(An,An+1,...)=nσ(Bn,Bn+1,...):=τBn

Isso é verdade?


Eu acho que existe uma função st são independentes, então podemos escolher . Isso é verdade? Porque porque não? Caso contrário, de que outra forma posso provar ou refutar a conjectura acima? Se for verdade, acho que pode ser provado modificando a prova da Lei Kolmogorov 0-1 (para eventos).f:NNAf(n)Bn=Af(n)


Talvez uma dessas subsequências de conjuntos seja independente:

An

A2n,A2n+1

A3n,A3n+1,A3n+2

Amn,Amn+1,Amn+2,...,Amn+(m1)

Acho que temos isso

τAn=τAmn+i:=nσ(Amn+i,Am(n+1)+i,...)

onde e . i { 0 , 1 , 2 , . . . , m - 1 }mNi{0,1,2,...,m1}


Parece que precisamos de qualquer , se existir, para satisfazer a seguinte condição:f(n)

(**)σ(Af(n),Af(n+1)...)σ(An,An+1,...)

o que eu acho que é verdade se (e somente se?)f(n)n .


Outros possíveis candidatos a :f(n) f : NN ( * * ) f ( n ) n (suponha que as variáveis ​​sejam st estão satisfeitas. Se necessário, ou também.)f:NN()f(n)n

  1. i=0maini

  2. 2n,3n,...

  3. i=1mbicin

  4. t > um e 1 / etn,tn ( acho quet>e1/e )

  5. i=1mbicin,i=1mbicin

  6. linear combination of trigonometric functions,linear combination of trigonometric functions

  7. Some linear combination of the above,Some linear combination of the above


Supondo que a conjectura seja verdadeira , acho que não é necessário encontrar que funcione para todas as sequências possíveis de eventos porque esse pode nem existir.A 1 , A 2 , . . . f ( n )f(n)A1,A2,...f(n)


Para refutar a conjectura : Acho que devemos mostrar que essa sequência sendo independente implica que tail nunca será igual a tail, pois tail será trivial por Kolmogorov 0-1 Law (para eventos).B n A n B n P -BnBnAnBnP

Algo que pode ajudar: podemos mostrar que ou e não são independentes, mas não tenho certeza de que a conjectura seja contestada porque poderia construir alguns 's que se parecem com:1 n N , A f ( n ) , um f ( n + 1 ) , . . . B n Anσ(Af(n),Af(n+1),...),P(A)=01nN,Af(n),Af(n+1),...Bn

  1. Bn=An+1An
  2. Bn=AnAn1,A0=
  3. Bn=mAmn
  4. Bn=mAmn
  5. B2n=mAmn,B2n+1=mAmn
  6. Bn=lim supmAmn
  7. Bn=lim infmAmn
  8. B2n=lim supmAmn,B2n+1=lim infmAmn

Sem dizer, é claro, que qualquer um desses satisfaz mas esse não precisa estar no formato .τ A n = τ B n B n A f ( n )BnτAn=τBnBnAf(n)


Borel-Cantelli:

  1. Se . Portanto, é independente.B m = lim sup Um m nnP(An)<0=P(lim supAn)=P(lim supAmn) mNBm=lim supAmn

  2. Se , talvez essa extensão do Borel-Cantelli ? Não tenho muita certeza se entendi ou como seria útil. Acho que não podemos concluir nada se tivermos .P ( lim sup Um n )nP(An)=P(lim supAn)

  3. Depois, há o caso de mas as condições anteriores não são satisfeitas.nP(An)=


1
Talvez uma prova por construção, onde ? B1=A1,B2=A2A1,
jbowman

1
Para mim, é improvável que essa conjectura seja verdadeira, a menos que você adicione condições extras ou queira dizer que as conclusões de dois álgebra concordam (o que ocorre quase trivialmente). No entanto, não consigo ver um exemplo contrário. σ
P.Windridge

1
De qualquer forma, acho que você pode começar com a pergunta (mais simples): "Seja um espaço de probabilidade. Suponha que é um álgebra gerado de forma contável e ou para qualquer evento . Existe uma sequência de eventos independentes in com cauda -algebra ?GF σ P ( A ) = 0 1 A G B 1 , B 2 , F σ G(Ω,F,P)GFσP(A)=01AGB1,B2,FσG
P.Windridge

2
Um álgebra é gerado de forma contável se existir st . É simples encontrar exemplos em que a cauda álgebra não é gerada de forma contável. G F 1 , F 2 , G = σ ( F 1 , F 2 , ) σσGF1,F2,G=σ(F1,F2,)σ
P.Windridge

2
De maneira mais geral, um sub- álgebra de um álgebra gerado de forma contável pode não ser ele próprio contado! Na verdade, veja o Exercício 1.1.18 em math.mit.edu/~dws/175/prob01.pdfσσσ
P.Windridge

Respostas:


7

Se você deseja que os eventos sejam independentes de uma maneira interessante (não simplesmente porque ou ), a conjectura é falsa.P ( B n ) = 0 P ( B n ) = 1BnP(Bn)=0P(Bn)=1

Aqui está um exemplo pedante. Suponha que seja um espaço de probabilidade adequadamente rico. (Ω,F,P)

Deixe ser -null, isto é . Tome , de modo que a cauda álgebra seja .P P ( A ) = 0 A i = A σ G = { , A , A c , Ω }AFPP(A)=0Ai=AσG={,A,Ac,Ω}

Note que em particular é finito.G

Agora, suponha que seja uma sequência independente de eventos com limitado a e . Então a cauda -algebra não é gerada de forma contável. (Veja, por exemplo, o Exercício 1.1.18 http://math.mit.edu/~dws/175/prob01.pdf , que usa um argumento como eu descrevi acima - qualquer -trivial -algebra gerado um átomo de massa , mas não possui esse átomo).P ( B n ) 0 1 σ H P σ 1 HB1,B2,P(Bn)01σHPσ1H

Portanto, é finito, mas nem é gerado de forma contável.HGH


Edição 2: se você aceitarP(Bn)=0P σ G E 1 , E 2 , G F G P E n E c n B B 1 , 1 = E 1 B 2 , 1 = E 1 , B 2 , 2 = E 2 ,, B k , j = E j , poderá replicar qualquer -trivial -algebra gerado de forma contável . Mais detalhadamente, suponha que seja gerado pelos eventos . Se for trivial, os serão todos independentes, em virtude de serem nulos (ou ser nulo). Agora faça uma construção triangular para os eventos : , , . PσGE1,E2,GFGPEnEncBB1,1=E1B2,1=E1,B2,2=E2,,Bk,j=Ej1jk

Então é uma sequência contável (com ordenação natural para os índices) de eventos independentes cuja cauda álgebra é .σ G(Bk,j)σG

Então, aqui eu acho que é a pergunta-chave: suponha que seja uma cauda trivial gerável de maneira contável álgebra (proveniente de eventos não nulos que possam ser dependentes). Pode ser realizado como a cauda -álgebra para alguns eventos nulos?P σ G σGPσGσ

Edit 1: Uma área cinza é o que acontece se você aceitar , embora esse não pareça ser o objetivo da pergunta original.P(Bn)0


Obrigado P.Windridge, mas não tenho certeza se entendi. 1 Se incluirmos ou , a conjectura é (trivialmente?) Verdadeira? 2 O que você está tentando provar no Edit 2? Se sim, seu igual a meu ? Eu editei OP para abreviação1 G τ A nP(Bn)=01GτAn
BCLC

Eu li o exercício. ? H=τBn
BCLC 30/12/2015

Oi BCLC, (1) Estou dizendo que, se incluirmos , a conjectura é verdadeira para todas as opções dos eventos que possuem uma cauda "agradável" -algebra (onde "bom" aqui significa gerado de forma contável). (2) Sim e é o seu . Nota: o exercício vinculado usa " " para o que deveria ser seu , e " " denota uma sequência candidata de eventos geradores (usada para obter uma contradição.A 1 , A 2 , σ G = τ ( A n ) H τ ( B n ) A n B n B nP(Bn)=0A1,A2,σG=τ(An)Hτ(Bn)AnBnBn
P.Windridge

Não tenho certeza se sigo. Apenas pelos pressupostos, os são independentes? Ai
BCLC 01/01

No Exercício 1.1.18 de math.mit.edu/~dws/175/prob01.pdf , os são eventos independentes, que você deve considerar os em sua conjectura. Era isso que você estava perguntando? B iAiBi
precisa saber é o seguinte
Ao utilizar nosso site, você reconhece que leu e compreendeu nossa Política de Cookies e nossa Política de Privacidade.
Licensed under cc by-sa 3.0 with attribution required.